• YOU can help the next generation of students in the community!
    Share your trial papers and notes on our Notes & Resources page

Mechanics Help! (1 Viewer)

jet

Banned
Joined
Jan 4, 2007
Messages
3,148
Gender
Male
HSC
2009
We know we'll need an expression for the terminal velocity, so we have to calculate it:


This makes sense - it should depend on the strength of gravity and the resistive force of the medium (i.e. )

The force on the second particle is when our axis goes vertically upwards from the origin. Hence,


The second particle is momentarily at rest when . This means


So when we substitute into the equation for velocity of the first particle (and writing for exponential), we get
 

jet

Banned
Joined
Jan 4, 2007
Messages
3,148
Gender
Male
HSC
2009
Thanks guys :)

for terminal velocity, we could also say acceleration =0 right?
That's the more physical way of interpreting it (and technically more correct). I just know this means it will have eventual constant speed so I take the limit, there's no difference.
 

jet

Banned
Joined
Jan 4, 2007
Messages
3,148
Gender
Male
HSC
2009
4 = 22 so you move one of the twos into the 2k, and 2*a > a when a is not zero or negative (a is 2k+1 in this case so it can never be 0 or negative).
 

jet

Banned
Joined
Jan 4, 2007
Messages
3,148
Gender
Male
HSC
2009
They're not... Can't you see the > sign?
 

barbernator

Active Member
Joined
Sep 13, 2010
Messages
1,439
Gender
Male
HSC
2012
I know that, but how could you write 2.2^k+1 as 2^k+1

shouldn't it be 2^k+2 ?
they arent writing it as that. They are saying that one is less than the other for the purpose of the proof.
 

nightweaver066

Well-Known Member
Joined
Jul 7, 2010
Messages
1,585
Gender
Male
HSC
2012
Suppose z = 2(cos@ + isin@)

Conjugate z = 2(cos@ - isin@), so just reflected across the x-axis.

-z/2 would be reflected across the x and the y axis where the modulus is halved (so now 2)

1/z - it is reflected across the x-axis and the modulus is now 1/4 (shrinked to 25% original)

sqrt(z) - modulus is now 2, and argument is halved (De Moivre's theorem).
 

Users Who Are Viewing This Thread (Users: 0, Guests: 1)

Top